Sufficient & Necessary Questions - - Question 38

If the economy is weak, then prices remain constant although unemployment rises. But unemployment rises only if inve...

Alex07 May 31, 2013

Help

Is there a video explanation for this question? If there isn't can someone explain how to arrive at the correct answer?

Replies
Create a free account to read and take part in forum discussions.

Already have an account? log in

Mehran May 31, 2013

Hey Alex,

Thanks for your question. We will have a video explanation for this question live by tomorrow, but in the meantime, I will explain it here.

Let's start by breaking down the stimulus.

1st Sentence:

"If the economy is weak, then prices remain constant although unemployment rises."

So . . .

EW ==> PC & UR
not UR or not PC ==> not EW

2nd Sentence:

"But unemployment rises only if investment decreases."

So . . .

UR ==> ID
not ID ==> not UR

Last Sentence:

"Fortunately, investment is not decreasing."

So . . . "not ID" exists.

This information allows us to conclude that "not UR" (because not ID ==> not UR) and "not EW" (because not UR ==> not EW; either not UR or not PC is sufficient for not EW according to the contrapositive of the 1st sentence).

Okay, turning to the question stem, we are looking for what "must be false" (or CANNOT BE TRUE). Since we have already concluded not UR and not EW, we would be looking for an answer choice that says UR or EW.

Answer Choice (A):

"Either the economy is weak or investment is decreasing."

For either one, pick a variable (EW or ID) and negate for the sufficient condition, and the other variable is your necessary condition. So . . .

not EW ==> ID
not ID ==> EW

We have concluded "not EW" from the stimulus and according to (A)'s principle, that would lead to the conclusion of "ID" (i.e. investment decreasing). However, the last sentence told us that investment is not decreasing, so (A) cannot be true and (A) would be the correct answer.

Mehran May 31, 2013

Answer Choice (B):

"If unemployment rises, the prices remain constant."

UR ==> PC
not PC ==> not UR

While we have concluded "not UR" from the information in the stimulus, not UR is the necessary condition in (B)'s principle, which tells us nothing (DON'T JUST NEGATE), so (B) could be true.

Mehran May 31, 2013

Answer Choice (C):

"The economy is weak only if investment decreases."

EW ==> ID
not ID ==> not EW

The stimulus already told us "not ID" which allowed us to conclude "not EW" so (C) must be true. Notice that (C) is the transitive property from the 1st and 2nd sentence (i.e. EW ==> UR ==> ID).

Mehran May 31, 2013

Answer Choice (D):

"Either the economy is weak or prices are remaining constant."

not EW ==> PC
not PC ==> EW

We have concluded "not EW" from the information in the stimulus, and according to (D)'s principle, that would allow us to conclude "PC." We know nothing about whether prices remained constant or not so (D) could be true.

Mehran May 31, 2013

Answer Choice (E):

"Either unemployment is rising or the economy is not weak."

not UR ==> not EW
EW ==> UR

This answer is a direct restatement of the 1st sentence (without any mention of PC). But remember, the necessary condition in the 1st sentence was a compound condition (i.e. both PC and UR are necessary for EW). As such, you could write these separately as follows:

EW ==> PC
EW ==> UR

So (E) must be true and, therefore, incorrect.

Hope this helps! We will have the video live tomorrow. Let us know if you have any other questions.

Mehran June 1, 2013

The video explanation for this question is now live inside of LSATMax. Please let us know if you have any other questions.

Batman September 6, 2015

According to the explanation of (D) above, although " we know nothing about whether price is constant or not," we, however, may conclude " notPC" from the information in the stimulus, which you stated "not PC - >not EW." However, according to the contrapositive of (D), it says " not PC - ->EW." Therefore, shouldn't we also pick (D) as a CANNOT BE TRUE???

Naz September 9, 2015

Remember that we have already deduced that we have "not EW." Therefore, the contrapositive of (D) is not applicable. Though (D) is saying we wither have a weak economy or prices remain constant, we know that we do not have a weak economy, which leaves us with: if we do not have a weak economy, we have prices remain constant. Well, this could be true. We have no information to say otherwise.

Thus, answer choice (D) is not the correct answer.

Whereas with answer choice (A), we are both told that investment is not decreasing, and we have deduced that the economy is not weak. Answer choice (A), therefore, in no way can be true since we have direct information to contradict it.

Hope that clears things up! Please let us know if you have any other questions.

alymathieu September 29, 2018

I'm confused as to why we would be looking for an answer choice hat says UR or EW. Is it because it is the contra positive of not UR and Not EW because I don't see why we would make a contrapositive for that when it's not stated in the premise and the premise actually says not PC or Not UR> not EE

yckim2180 February 6, 2020

I've read all the responses from the instructors on why D can be true, but it's still unclear. Given that the reasoning structure of D parallels A, it's difficult to understand how D can be true when A cannot.

A. not EW - -> ID
not ID - -> EW (directly contradicts not ID - > not EW (not ID - > not UR - > not EW))
D. not EW - > PC
not PC - > EW (directly contradicts not PC - > not EW)

Brett-Lindsay July 11, 2020

I read D as being
not EW --> PC
not PC --> EW

but our stimulus tells us:
EW --> PC & UR
not PC --> not EW
(removing the "or not UR" from not PC or not UR --> not EW)

From the stimulus, we don't have any information at all about the situation for "not EW," so it could be possible that even though we have "not EW," we might still be able to have "PC"

I think that's why D is not the answer, but it is not necessarily false (could be true), rather than must be false (cannot be true).

Victoria July 15, 2020

Hi @alymathieu, @yckim2180, and @Brett-Lindsay,

Thanks for your questions! Hopefully I can help to clear things up a bit.

Answer choice (A) must be false because of the conclusion drawn in the stimulus. We know that:

(a) If the economy is weak, unemployment rises.
(b) If unemployment rises, investment decreases.

EW --> UR --> ID
Not ID --> Not UR --> Not EW

We also learn that investment is not decreasing. Therefore, we are focusing on our contrapositive chain.

Answer choice (A) cannot be true because neither element of the either/or statement can be true based on the information provided to us in the passage. We know that investment is not decreasing; therefore, the economy is not weak.

Answer choice (D) is a little bit more confusing. "Either the economy is weak or prices remain constant."

Not EW --> PC
Not PC --> EW

We know from the stimulus that:

EW --> PC
Not PC --> Not EW

@Brett-Lindsay explained it correctly! We do not have enough information from the stimulus to draw any conclusions regarding prices from the fact that the economy is not weak. To do so would be to mistakenly reverse our S&N diagram.

We know that prices will remain constant if the economy is weak; however, it is entirely possible that prices could also remain constant if the economy is strong. We just don't have enough information to definitively conclude that this is false.

On the other hand, answer choice (A) is directly contradicted by the stimulus.

Hope this is helpful! Keep up the good work and please let us know if you have any further questions.